Proof of polynomial divisibility without using complex numbers?












2














My question is the same as polynomial of degree n and its divisor except I want a solution that does not make use of complex numbers




Problem: Find all positive integers $n$ such that $x^2+x+1mid (x+1)^n+x^n+1$




Using wolframalpha, I can see that a number of solutions work, such as $2,4,7$. I tried substituting various values of $x$ in and then working in specific mod cases, but this doesn't really seem to work because it only gives possible values, not 'actual' values. Polynomial long division here is also quite unwieldy...










share|cite|improve this question
























  • $1$ and $7$ do not satisfy the condition.
    – Peter
    2 hours ago










  • @Peter You're right... I misread my own question! Thank you.
    – user574848
    2 hours ago
















2














My question is the same as polynomial of degree n and its divisor except I want a solution that does not make use of complex numbers




Problem: Find all positive integers $n$ such that $x^2+x+1mid (x+1)^n+x^n+1$




Using wolframalpha, I can see that a number of solutions work, such as $2,4,7$. I tried substituting various values of $x$ in and then working in specific mod cases, but this doesn't really seem to work because it only gives possible values, not 'actual' values. Polynomial long division here is also quite unwieldy...










share|cite|improve this question
























  • $1$ and $7$ do not satisfy the condition.
    – Peter
    2 hours ago










  • @Peter You're right... I misread my own question! Thank you.
    – user574848
    2 hours ago














2












2








2


1





My question is the same as polynomial of degree n and its divisor except I want a solution that does not make use of complex numbers




Problem: Find all positive integers $n$ such that $x^2+x+1mid (x+1)^n+x^n+1$




Using wolframalpha, I can see that a number of solutions work, such as $2,4,7$. I tried substituting various values of $x$ in and then working in specific mod cases, but this doesn't really seem to work because it only gives possible values, not 'actual' values. Polynomial long division here is also quite unwieldy...










share|cite|improve this question















My question is the same as polynomial of degree n and its divisor except I want a solution that does not make use of complex numbers




Problem: Find all positive integers $n$ such that $x^2+x+1mid (x+1)^n+x^n+1$




Using wolframalpha, I can see that a number of solutions work, such as $2,4,7$. I tried substituting various values of $x$ in and then working in specific mod cases, but this doesn't really seem to work because it only gives possible values, not 'actual' values. Polynomial long division here is also quite unwieldy...







polynomials divisibility






share|cite|improve this question















share|cite|improve this question













share|cite|improve this question




share|cite|improve this question








edited 2 hours ago

























asked 3 hours ago









user574848

16014




16014












  • $1$ and $7$ do not satisfy the condition.
    – Peter
    2 hours ago










  • @Peter You're right... I misread my own question! Thank you.
    – user574848
    2 hours ago


















  • $1$ and $7$ do not satisfy the condition.
    – Peter
    2 hours ago










  • @Peter You're right... I misread my own question! Thank you.
    – user574848
    2 hours ago
















$1$ and $7$ do not satisfy the condition.
– Peter
2 hours ago




$1$ and $7$ do not satisfy the condition.
– Peter
2 hours ago












@Peter You're right... I misread my own question! Thank you.
– user574848
2 hours ago




@Peter You're right... I misread my own question! Thank you.
– user574848
2 hours ago










3 Answers
3






active

oldest

votes


















2














Since $$(x+1)^6equiv 1mod (x^2+x+1)$$ and $$x^6equiv 1mod (x^2+x+1)$$ the exponent $n$ can be reduced modulo $6$. Inspection gives the solutions $2$ and $4$ in the interval $[1,6]$, hence the condition is satisfied if and only if $nequiv pm2mod 6$. So exactly the even exponents $n$ not divisible by $3$ do the job.






share|cite|improve this answer





















  • How does one see the initial congruence?
    – user574848
    2 hours ago










  • The second one is easier to find : $x^3-1=(x^2+x+1)(x-1)$
    – Peter
    2 hours ago










  • For the first, consider $$(x+1)^2=x^2+2x+1equiv xmod (x^2+x+1)$$
    – Peter
    2 hours ago





















1














You can use a form of polynomial modular arithmetic here, working modulo $x^2+x+1$



We have $x^2+x+1equiv 0$ so that $x+1equiv -x^2$



also $x^3+x^2+xequiv 0$ so that $x^3equiv -x^2-xequiv 1$



So $p_n(x)=(x+1)^n+x^n+1equiv (-1)^nx^{2n}+x^n+1$



Now since $x^3equiv 1$ and you have a $(-1)^n$ there you can work modulo $6$, because extra multiples of $6$ in the powers change nothing.



We have $$p_0(x)equiv 3$$ $$p_1(x)equiv -x^2+x+1equiv 2x+2$$$$p_2(x)equiv x^4+x^2+1equiv x+x^2+1equiv 0$$$$p_3(x)equiv -x^6+x^3+1equiv 1$$$$p_4(x)equiv x^8+x^4+1equiv x^2+x+1equiv 0$$$$p_5(x)=-x^{10}+x^5+1equiv-x+x^2+1equiv-2x$$



So the division works for $nequiv 2,4 bmod 6$ and you have the remainders on polynomial division for the other residue classes.





Note that because $x^2+x+1$ is an irreducible polynomial, this method is algebraically equivalent to replacing $x$ by a root of the equation $x^2+x+1$, but the computations all take place in a polynomial ring which isn't identified with any particular known ring or field.






share|cite|improve this answer































    0














    First I will prove that: $$n in {2,4} pmod{6}$$ is a necessary condition:



    Just substitute x for the value 2 so that the equation results:



    $$7 | 3 ^ n + 2 ^ n + 1$$



    $$3 ^ n + 2 ^ n + 1 equiv 0 pmod{7}$$



    whose all solutions are:
    $$n in {2,4} pmod{6}$$



    Now I am going to prove that these two conditions are sufficient:



    As mentioned above:
    $$ P = x^2+x+1 $$
    $$(x + 1) ^ 6 equiv 1 pmod{P}$$



    $$x ^ 6 = 1 pmod{P}$$



    If $n equiv 2 pmod {6}$:



    $$(x + 1) ^ {6a+2} + x ^ {6a+2} + 1 pmod{P}$$
    $$(x + 1) ^ {6a} (x ^ 2 + 2x + 1) + x ^ {6a} x ^ 2 + 1 pmod{P}$$
    $$(P + x) + x ^ 2 + 1 pmod{P}$$
    $$P + x + x ^ 2 +1 pmod{P}$$
    $$2P pmod{P}$$
    $$ 0 pmod{P} $$



    With which is a sufficient condition.



    If $n equiv 4 pmod {6}$:



    $$(x + 1) ^ {6a+4} + x ^ {6a+4} + 1 pmod{P}$$
    $$(x + 1) ^ {6a} (x ^ 4 + 4x ^ 3 + 6x ^ 2 + 4x + 1) + x ^ {6a} x ^ 4 + 1 pmod{P}$$
    $$(x ^ 4 + 4x ^ 3 + 6x ^ 2 + 4x + 1) + x ^ 4 + 1 pmod{P}$$
    $$2x^4 +4x^3+6x ^ 2 + 4x + 2 pmod{P}$$
    $$2(x^2+x+1)^2 pmod{P}$$
    $$ 0 pmod{P} $$



    With which is a sufficient condition.



    It is proved that the condition is necessary and sufficient.






    share|cite|improve this answer























      Your Answer





      StackExchange.ifUsing("editor", function () {
      return StackExchange.using("mathjaxEditing", function () {
      StackExchange.MarkdownEditor.creationCallbacks.add(function (editor, postfix) {
      StackExchange.mathjaxEditing.prepareWmdForMathJax(editor, postfix, [["$", "$"], ["\\(","\\)"]]);
      });
      });
      }, "mathjax-editing");

      StackExchange.ready(function() {
      var channelOptions = {
      tags: "".split(" "),
      id: "69"
      };
      initTagRenderer("".split(" "), "".split(" "), channelOptions);

      StackExchange.using("externalEditor", function() {
      // Have to fire editor after snippets, if snippets enabled
      if (StackExchange.settings.snippets.snippetsEnabled) {
      StackExchange.using("snippets", function() {
      createEditor();
      });
      }
      else {
      createEditor();
      }
      });

      function createEditor() {
      StackExchange.prepareEditor({
      heartbeatType: 'answer',
      autoActivateHeartbeat: false,
      convertImagesToLinks: true,
      noModals: true,
      showLowRepImageUploadWarning: true,
      reputationToPostImages: 10,
      bindNavPrevention: true,
      postfix: "",
      imageUploader: {
      brandingHtml: "Powered by u003ca class="icon-imgur-white" href="https://imgur.com/"u003eu003c/au003e",
      contentPolicyHtml: "User contributions licensed under u003ca href="https://creativecommons.org/licenses/by-sa/3.0/"u003ecc by-sa 3.0 with attribution requiredu003c/au003e u003ca href="https://stackoverflow.com/legal/content-policy"u003e(content policy)u003c/au003e",
      allowUrls: true
      },
      noCode: true, onDemand: true,
      discardSelector: ".discard-answer"
      ,immediatelyShowMarkdownHelp:true
      });


      }
      });














      draft saved

      draft discarded


















      StackExchange.ready(
      function () {
      StackExchange.openid.initPostLogin('.new-post-login', 'https%3a%2f%2fmath.stackexchange.com%2fquestions%2f3054769%2fproof-of-polynomial-divisibility-without-using-complex-numbers%23new-answer', 'question_page');
      }
      );

      Post as a guest















      Required, but never shown

























      3 Answers
      3






      active

      oldest

      votes








      3 Answers
      3






      active

      oldest

      votes









      active

      oldest

      votes






      active

      oldest

      votes









      2














      Since $$(x+1)^6equiv 1mod (x^2+x+1)$$ and $$x^6equiv 1mod (x^2+x+1)$$ the exponent $n$ can be reduced modulo $6$. Inspection gives the solutions $2$ and $4$ in the interval $[1,6]$, hence the condition is satisfied if and only if $nequiv pm2mod 6$. So exactly the even exponents $n$ not divisible by $3$ do the job.






      share|cite|improve this answer





















      • How does one see the initial congruence?
        – user574848
        2 hours ago










      • The second one is easier to find : $x^3-1=(x^2+x+1)(x-1)$
        – Peter
        2 hours ago










      • For the first, consider $$(x+1)^2=x^2+2x+1equiv xmod (x^2+x+1)$$
        – Peter
        2 hours ago


















      2














      Since $$(x+1)^6equiv 1mod (x^2+x+1)$$ and $$x^6equiv 1mod (x^2+x+1)$$ the exponent $n$ can be reduced modulo $6$. Inspection gives the solutions $2$ and $4$ in the interval $[1,6]$, hence the condition is satisfied if and only if $nequiv pm2mod 6$. So exactly the even exponents $n$ not divisible by $3$ do the job.






      share|cite|improve this answer





















      • How does one see the initial congruence?
        – user574848
        2 hours ago










      • The second one is easier to find : $x^3-1=(x^2+x+1)(x-1)$
        – Peter
        2 hours ago










      • For the first, consider $$(x+1)^2=x^2+2x+1equiv xmod (x^2+x+1)$$
        – Peter
        2 hours ago
















      2












      2








      2






      Since $$(x+1)^6equiv 1mod (x^2+x+1)$$ and $$x^6equiv 1mod (x^2+x+1)$$ the exponent $n$ can be reduced modulo $6$. Inspection gives the solutions $2$ and $4$ in the interval $[1,6]$, hence the condition is satisfied if and only if $nequiv pm2mod 6$. So exactly the even exponents $n$ not divisible by $3$ do the job.






      share|cite|improve this answer












      Since $$(x+1)^6equiv 1mod (x^2+x+1)$$ and $$x^6equiv 1mod (x^2+x+1)$$ the exponent $n$ can be reduced modulo $6$. Inspection gives the solutions $2$ and $4$ in the interval $[1,6]$, hence the condition is satisfied if and only if $nequiv pm2mod 6$. So exactly the even exponents $n$ not divisible by $3$ do the job.







      share|cite|improve this answer












      share|cite|improve this answer



      share|cite|improve this answer










      answered 2 hours ago









      Peter

      46.6k1039125




      46.6k1039125












      • How does one see the initial congruence?
        – user574848
        2 hours ago










      • The second one is easier to find : $x^3-1=(x^2+x+1)(x-1)$
        – Peter
        2 hours ago










      • For the first, consider $$(x+1)^2=x^2+2x+1equiv xmod (x^2+x+1)$$
        – Peter
        2 hours ago




















      • How does one see the initial congruence?
        – user574848
        2 hours ago










      • The second one is easier to find : $x^3-1=(x^2+x+1)(x-1)$
        – Peter
        2 hours ago










      • For the first, consider $$(x+1)^2=x^2+2x+1equiv xmod (x^2+x+1)$$
        – Peter
        2 hours ago


















      How does one see the initial congruence?
      – user574848
      2 hours ago




      How does one see the initial congruence?
      – user574848
      2 hours ago












      The second one is easier to find : $x^3-1=(x^2+x+1)(x-1)$
      – Peter
      2 hours ago




      The second one is easier to find : $x^3-1=(x^2+x+1)(x-1)$
      – Peter
      2 hours ago












      For the first, consider $$(x+1)^2=x^2+2x+1equiv xmod (x^2+x+1)$$
      – Peter
      2 hours ago






      For the first, consider $$(x+1)^2=x^2+2x+1equiv xmod (x^2+x+1)$$
      – Peter
      2 hours ago













      1














      You can use a form of polynomial modular arithmetic here, working modulo $x^2+x+1$



      We have $x^2+x+1equiv 0$ so that $x+1equiv -x^2$



      also $x^3+x^2+xequiv 0$ so that $x^3equiv -x^2-xequiv 1$



      So $p_n(x)=(x+1)^n+x^n+1equiv (-1)^nx^{2n}+x^n+1$



      Now since $x^3equiv 1$ and you have a $(-1)^n$ there you can work modulo $6$, because extra multiples of $6$ in the powers change nothing.



      We have $$p_0(x)equiv 3$$ $$p_1(x)equiv -x^2+x+1equiv 2x+2$$$$p_2(x)equiv x^4+x^2+1equiv x+x^2+1equiv 0$$$$p_3(x)equiv -x^6+x^3+1equiv 1$$$$p_4(x)equiv x^8+x^4+1equiv x^2+x+1equiv 0$$$$p_5(x)=-x^{10}+x^5+1equiv-x+x^2+1equiv-2x$$



      So the division works for $nequiv 2,4 bmod 6$ and you have the remainders on polynomial division for the other residue classes.





      Note that because $x^2+x+1$ is an irreducible polynomial, this method is algebraically equivalent to replacing $x$ by a root of the equation $x^2+x+1$, but the computations all take place in a polynomial ring which isn't identified with any particular known ring or field.






      share|cite|improve this answer




























        1














        You can use a form of polynomial modular arithmetic here, working modulo $x^2+x+1$



        We have $x^2+x+1equiv 0$ so that $x+1equiv -x^2$



        also $x^3+x^2+xequiv 0$ so that $x^3equiv -x^2-xequiv 1$



        So $p_n(x)=(x+1)^n+x^n+1equiv (-1)^nx^{2n}+x^n+1$



        Now since $x^3equiv 1$ and you have a $(-1)^n$ there you can work modulo $6$, because extra multiples of $6$ in the powers change nothing.



        We have $$p_0(x)equiv 3$$ $$p_1(x)equiv -x^2+x+1equiv 2x+2$$$$p_2(x)equiv x^4+x^2+1equiv x+x^2+1equiv 0$$$$p_3(x)equiv -x^6+x^3+1equiv 1$$$$p_4(x)equiv x^8+x^4+1equiv x^2+x+1equiv 0$$$$p_5(x)=-x^{10}+x^5+1equiv-x+x^2+1equiv-2x$$



        So the division works for $nequiv 2,4 bmod 6$ and you have the remainders on polynomial division for the other residue classes.





        Note that because $x^2+x+1$ is an irreducible polynomial, this method is algebraically equivalent to replacing $x$ by a root of the equation $x^2+x+1$, but the computations all take place in a polynomial ring which isn't identified with any particular known ring or field.






        share|cite|improve this answer


























          1












          1








          1






          You can use a form of polynomial modular arithmetic here, working modulo $x^2+x+1$



          We have $x^2+x+1equiv 0$ so that $x+1equiv -x^2$



          also $x^3+x^2+xequiv 0$ so that $x^3equiv -x^2-xequiv 1$



          So $p_n(x)=(x+1)^n+x^n+1equiv (-1)^nx^{2n}+x^n+1$



          Now since $x^3equiv 1$ and you have a $(-1)^n$ there you can work modulo $6$, because extra multiples of $6$ in the powers change nothing.



          We have $$p_0(x)equiv 3$$ $$p_1(x)equiv -x^2+x+1equiv 2x+2$$$$p_2(x)equiv x^4+x^2+1equiv x+x^2+1equiv 0$$$$p_3(x)equiv -x^6+x^3+1equiv 1$$$$p_4(x)equiv x^8+x^4+1equiv x^2+x+1equiv 0$$$$p_5(x)=-x^{10}+x^5+1equiv-x+x^2+1equiv-2x$$



          So the division works for $nequiv 2,4 bmod 6$ and you have the remainders on polynomial division for the other residue classes.





          Note that because $x^2+x+1$ is an irreducible polynomial, this method is algebraically equivalent to replacing $x$ by a root of the equation $x^2+x+1$, but the computations all take place in a polynomial ring which isn't identified with any particular known ring or field.






          share|cite|improve this answer














          You can use a form of polynomial modular arithmetic here, working modulo $x^2+x+1$



          We have $x^2+x+1equiv 0$ so that $x+1equiv -x^2$



          also $x^3+x^2+xequiv 0$ so that $x^3equiv -x^2-xequiv 1$



          So $p_n(x)=(x+1)^n+x^n+1equiv (-1)^nx^{2n}+x^n+1$



          Now since $x^3equiv 1$ and you have a $(-1)^n$ there you can work modulo $6$, because extra multiples of $6$ in the powers change nothing.



          We have $$p_0(x)equiv 3$$ $$p_1(x)equiv -x^2+x+1equiv 2x+2$$$$p_2(x)equiv x^4+x^2+1equiv x+x^2+1equiv 0$$$$p_3(x)equiv -x^6+x^3+1equiv 1$$$$p_4(x)equiv x^8+x^4+1equiv x^2+x+1equiv 0$$$$p_5(x)=-x^{10}+x^5+1equiv-x+x^2+1equiv-2x$$



          So the division works for $nequiv 2,4 bmod 6$ and you have the remainders on polynomial division for the other residue classes.





          Note that because $x^2+x+1$ is an irreducible polynomial, this method is algebraically equivalent to replacing $x$ by a root of the equation $x^2+x+1$, but the computations all take place in a polynomial ring which isn't identified with any particular known ring or field.







          share|cite|improve this answer














          share|cite|improve this answer



          share|cite|improve this answer








          edited 2 hours ago

























          answered 2 hours ago









          Mark Bennet

          80.3k981179




          80.3k981179























              0














              First I will prove that: $$n in {2,4} pmod{6}$$ is a necessary condition:



              Just substitute x for the value 2 so that the equation results:



              $$7 | 3 ^ n + 2 ^ n + 1$$



              $$3 ^ n + 2 ^ n + 1 equiv 0 pmod{7}$$



              whose all solutions are:
              $$n in {2,4} pmod{6}$$



              Now I am going to prove that these two conditions are sufficient:



              As mentioned above:
              $$ P = x^2+x+1 $$
              $$(x + 1) ^ 6 equiv 1 pmod{P}$$



              $$x ^ 6 = 1 pmod{P}$$



              If $n equiv 2 pmod {6}$:



              $$(x + 1) ^ {6a+2} + x ^ {6a+2} + 1 pmod{P}$$
              $$(x + 1) ^ {6a} (x ^ 2 + 2x + 1) + x ^ {6a} x ^ 2 + 1 pmod{P}$$
              $$(P + x) + x ^ 2 + 1 pmod{P}$$
              $$P + x + x ^ 2 +1 pmod{P}$$
              $$2P pmod{P}$$
              $$ 0 pmod{P} $$



              With which is a sufficient condition.



              If $n equiv 4 pmod {6}$:



              $$(x + 1) ^ {6a+4} + x ^ {6a+4} + 1 pmod{P}$$
              $$(x + 1) ^ {6a} (x ^ 4 + 4x ^ 3 + 6x ^ 2 + 4x + 1) + x ^ {6a} x ^ 4 + 1 pmod{P}$$
              $$(x ^ 4 + 4x ^ 3 + 6x ^ 2 + 4x + 1) + x ^ 4 + 1 pmod{P}$$
              $$2x^4 +4x^3+6x ^ 2 + 4x + 2 pmod{P}$$
              $$2(x^2+x+1)^2 pmod{P}$$
              $$ 0 pmod{P} $$



              With which is a sufficient condition.



              It is proved that the condition is necessary and sufficient.






              share|cite|improve this answer




























                0














                First I will prove that: $$n in {2,4} pmod{6}$$ is a necessary condition:



                Just substitute x for the value 2 so that the equation results:



                $$7 | 3 ^ n + 2 ^ n + 1$$



                $$3 ^ n + 2 ^ n + 1 equiv 0 pmod{7}$$



                whose all solutions are:
                $$n in {2,4} pmod{6}$$



                Now I am going to prove that these two conditions are sufficient:



                As mentioned above:
                $$ P = x^2+x+1 $$
                $$(x + 1) ^ 6 equiv 1 pmod{P}$$



                $$x ^ 6 = 1 pmod{P}$$



                If $n equiv 2 pmod {6}$:



                $$(x + 1) ^ {6a+2} + x ^ {6a+2} + 1 pmod{P}$$
                $$(x + 1) ^ {6a} (x ^ 2 + 2x + 1) + x ^ {6a} x ^ 2 + 1 pmod{P}$$
                $$(P + x) + x ^ 2 + 1 pmod{P}$$
                $$P + x + x ^ 2 +1 pmod{P}$$
                $$2P pmod{P}$$
                $$ 0 pmod{P} $$



                With which is a sufficient condition.



                If $n equiv 4 pmod {6}$:



                $$(x + 1) ^ {6a+4} + x ^ {6a+4} + 1 pmod{P}$$
                $$(x + 1) ^ {6a} (x ^ 4 + 4x ^ 3 + 6x ^ 2 + 4x + 1) + x ^ {6a} x ^ 4 + 1 pmod{P}$$
                $$(x ^ 4 + 4x ^ 3 + 6x ^ 2 + 4x + 1) + x ^ 4 + 1 pmod{P}$$
                $$2x^4 +4x^3+6x ^ 2 + 4x + 2 pmod{P}$$
                $$2(x^2+x+1)^2 pmod{P}$$
                $$ 0 pmod{P} $$



                With which is a sufficient condition.



                It is proved that the condition is necessary and sufficient.






                share|cite|improve this answer


























                  0












                  0








                  0






                  First I will prove that: $$n in {2,4} pmod{6}$$ is a necessary condition:



                  Just substitute x for the value 2 so that the equation results:



                  $$7 | 3 ^ n + 2 ^ n + 1$$



                  $$3 ^ n + 2 ^ n + 1 equiv 0 pmod{7}$$



                  whose all solutions are:
                  $$n in {2,4} pmod{6}$$



                  Now I am going to prove that these two conditions are sufficient:



                  As mentioned above:
                  $$ P = x^2+x+1 $$
                  $$(x + 1) ^ 6 equiv 1 pmod{P}$$



                  $$x ^ 6 = 1 pmod{P}$$



                  If $n equiv 2 pmod {6}$:



                  $$(x + 1) ^ {6a+2} + x ^ {6a+2} + 1 pmod{P}$$
                  $$(x + 1) ^ {6a} (x ^ 2 + 2x + 1) + x ^ {6a} x ^ 2 + 1 pmod{P}$$
                  $$(P + x) + x ^ 2 + 1 pmod{P}$$
                  $$P + x + x ^ 2 +1 pmod{P}$$
                  $$2P pmod{P}$$
                  $$ 0 pmod{P} $$



                  With which is a sufficient condition.



                  If $n equiv 4 pmod {6}$:



                  $$(x + 1) ^ {6a+4} + x ^ {6a+4} + 1 pmod{P}$$
                  $$(x + 1) ^ {6a} (x ^ 4 + 4x ^ 3 + 6x ^ 2 + 4x + 1) + x ^ {6a} x ^ 4 + 1 pmod{P}$$
                  $$(x ^ 4 + 4x ^ 3 + 6x ^ 2 + 4x + 1) + x ^ 4 + 1 pmod{P}$$
                  $$2x^4 +4x^3+6x ^ 2 + 4x + 2 pmod{P}$$
                  $$2(x^2+x+1)^2 pmod{P}$$
                  $$ 0 pmod{P} $$



                  With which is a sufficient condition.



                  It is proved that the condition is necessary and sufficient.






                  share|cite|improve this answer














                  First I will prove that: $$n in {2,4} pmod{6}$$ is a necessary condition:



                  Just substitute x for the value 2 so that the equation results:



                  $$7 | 3 ^ n + 2 ^ n + 1$$



                  $$3 ^ n + 2 ^ n + 1 equiv 0 pmod{7}$$



                  whose all solutions are:
                  $$n in {2,4} pmod{6}$$



                  Now I am going to prove that these two conditions are sufficient:



                  As mentioned above:
                  $$ P = x^2+x+1 $$
                  $$(x + 1) ^ 6 equiv 1 pmod{P}$$



                  $$x ^ 6 = 1 pmod{P}$$



                  If $n equiv 2 pmod {6}$:



                  $$(x + 1) ^ {6a+2} + x ^ {6a+2} + 1 pmod{P}$$
                  $$(x + 1) ^ {6a} (x ^ 2 + 2x + 1) + x ^ {6a} x ^ 2 + 1 pmod{P}$$
                  $$(P + x) + x ^ 2 + 1 pmod{P}$$
                  $$P + x + x ^ 2 +1 pmod{P}$$
                  $$2P pmod{P}$$
                  $$ 0 pmod{P} $$



                  With which is a sufficient condition.



                  If $n equiv 4 pmod {6}$:



                  $$(x + 1) ^ {6a+4} + x ^ {6a+4} + 1 pmod{P}$$
                  $$(x + 1) ^ {6a} (x ^ 4 + 4x ^ 3 + 6x ^ 2 + 4x + 1) + x ^ {6a} x ^ 4 + 1 pmod{P}$$
                  $$(x ^ 4 + 4x ^ 3 + 6x ^ 2 + 4x + 1) + x ^ 4 + 1 pmod{P}$$
                  $$2x^4 +4x^3+6x ^ 2 + 4x + 2 pmod{P}$$
                  $$2(x^2+x+1)^2 pmod{P}$$
                  $$ 0 pmod{P} $$



                  With which is a sufficient condition.



                  It is proved that the condition is necessary and sufficient.







                  share|cite|improve this answer














                  share|cite|improve this answer



                  share|cite|improve this answer








                  edited 10 mins ago

























                  answered 24 mins ago









                  Angel Moreno

                  35915




                  35915






























                      draft saved

                      draft discarded




















































                      Thanks for contributing an answer to Mathematics Stack Exchange!


                      • Please be sure to answer the question. Provide details and share your research!

                      But avoid



                      • Asking for help, clarification, or responding to other answers.

                      • Making statements based on opinion; back them up with references or personal experience.


                      Use MathJax to format equations. MathJax reference.


                      To learn more, see our tips on writing great answers.





                      Some of your past answers have not been well-received, and you're in danger of being blocked from answering.


                      Please pay close attention to the following guidance:


                      • Please be sure to answer the question. Provide details and share your research!

                      But avoid



                      • Asking for help, clarification, or responding to other answers.

                      • Making statements based on opinion; back them up with references or personal experience.


                      To learn more, see our tips on writing great answers.




                      draft saved


                      draft discarded














                      StackExchange.ready(
                      function () {
                      StackExchange.openid.initPostLogin('.new-post-login', 'https%3a%2f%2fmath.stackexchange.com%2fquestions%2f3054769%2fproof-of-polynomial-divisibility-without-using-complex-numbers%23new-answer', 'question_page');
                      }
                      );

                      Post as a guest















                      Required, but never shown





















































                      Required, but never shown














                      Required, but never shown












                      Required, but never shown







                      Required, but never shown

































                      Required, but never shown














                      Required, but never shown












                      Required, but never shown







                      Required, but never shown







                      Popular posts from this blog

                      サソリ

                      広島県道265号伴広島線

                      Accessing regular linux commands in Huawei's Dopra Linux